I bet U miss CR 5

This topic has expert replies
Legendary Member
Posts: 2326
Joined: Mon Jul 28, 2008 3:54 am
Thanked: 173 times
Followed by:2 members
GMAT Score:710

I bet U miss CR 5

by gmatmachoman » Mon Nov 09, 2009 7:53 am
Most people believe that yawning is most powerfully triggered by seeing someone else yawn. This belief about yawning is widespread not only today, but also has been commonplace in many parts of the world in the past, if we are to believe historians of popular culture. Thus, seeing someone else yawn must be the most irresistible cause of yawning.
The argument is most vulnerable to which one of the following criticisms?
(A) It attempts to support its conclusion solely by restating that conclusion in other words.
(B) It cites the evidence of historians of popular culture in direct support of a claim that lies outside their area of expertise.
(C) It makes a sweeping generalization about yawning based on evidence drawn from a limited number of atypical cases.
(D) It supports its conclusion by appealing solely to opinion in a matter that is largely factual.
(E) It takes for granted that yawns have no cause other than the one it cites.
18.

Master | Next Rank: 500 Posts
Posts: 199
Joined: Sat Oct 24, 2009 4:43 pm
Thanked: 22 times
GMAT Score:710

Re: I bet U miss CR 5

by palvarez » Mon Nov 09, 2009 8:09 am
D is the answer.

The truth value of X doesn't depend upon the the truth value of "I believe that X". Thats why "that" is called intentional predicate.

Legendary Member
Posts: 1578
Joined: Sun Dec 28, 2008 1:49 am
Thanked: 82 times
Followed by:9 members
GMAT Score:720

by maihuna » Mon Nov 09, 2009 9:47 am
yep it should be D.
Charged up again to beat the beast :)

Legendary Member
Posts: 1161
Joined: Mon May 12, 2008 2:52 am
Location: Sydney
Thanked: 23 times
Followed by:1 members

by mehravikas » Mon Nov 09, 2009 11:52 am
Agree that 'D' is the answer. But can anyone explain the reason to rule out 'E'?

User avatar
GMAT Instructor
Posts: 578
Joined: Tue Aug 25, 2009 6:00 pm
Thanked: 136 times
Followed by:62 members

by KapTeacherEli » Mon Nov 09, 2009 8:52 pm
mehravikas wrote:Agree that 'D' is the answer. But can anyone explain the reason to rule out 'E'?
Because the conclusion is about the "most irresistable" cause, so he is not in fact assuming there are no other causes--just weaker ones.
Eli Meyer
Kaplan GMAT Teacher
Cambridge, MA
www.kaptest.com/gmat

ImageImageImage

Master | Next Rank: 500 Posts
Posts: 200
Joined: Sat Aug 22, 2015 10:27 am

by james33 » Sun May 15, 2016 10:17 pm
I will Go with option D in this case.